LSAT and Law School Admissions Forum

Get expert LSAT preparation and law school admissions advice from PowerScore Test Preparation.

 Administrator
PowerScore Staff
  • PowerScore Staff
  • Posts: 8916
  • Joined: Feb 02, 2011
|
#22911
Complete Question Explanation

Justify the Conclusion. The correct answer choice is (A)

The conclusion in this stimulus is that insurance company regulations which deny coverage for lab tests decrease the quality of health care. This conclusion requires a couple assumptions. First, that the insurance restrictions are leading to reduced use of tests. (This seems like a plausible assumption according to ordinary economic reasoning, but it is not stated in the stimulus.) Second, that the lab tests actually improve the quality of health care.

The question stem asks us to "justify" the conclusion, meaning that a single correct answer choice will fill in all the missing assumptions and make the argument air-tight.

Answer choice (A): This is the correct answer choice. This answer choice states that the lab tests improve quality of medical care. This is the best answer choice, although it does not exactly "justify" the conclusion as the question stem asks. Note that the argument still has a missing assumption: it has failed to establish that the insurance restrictions are reducing use of the tests by patients. The LSAT test makers did not choose their words very carefully here.

Answer choice (B): The personal opinion of physicians regarding the insurance restrictions does not give us a clear implication about whether these lab tests improve quality of care. Perhaps physicians favor these tests because they truly provide value for patients \96 or perhaps physicians favor these tests because they are good revenue generators.

Answer choice (C): If some patients who might benefit from the tests have no health insurance whatsoever, then the insurance restrictions do not affect these patients. A reduction of quality of care is thus impossible for these patients (unless we make the roundabout argument that doctors lose revenue and thus the overall quality of their medical practice suffers). This answer choice tends to weaken the stimulus argument.

Answer choice (D): Even if "some" illnesses can be diagnosed without lab tests, this answer choice does not speak to the issue of whether lab tests improve quality of medical care. If anything, this answer choice weakens the stimulus by showing that lab tests are not quite so valuable as we might suppose.

Answer choice (E): This answer choice does not speak to the issues of (1) value added for patients by the lab tests, or (2) whether the insurance restrictions are reducing use of the lab tests.
 lsatstudier
  • Posts: 49
  • Joined: Oct 24, 2016
|
#30141
Hi,

In completing this question, I was able to identify my conclusion, but I'm starting to think I'm missing something in going from the conclusion to finding my answer choice. Any advice for prephrasing? Why is the answer A as opposed to E? Is it possible to set this up the way the book does such that A :arrow: B, etc.

Thank you!
 Claire Horan
PowerScore Staff
  • PowerScore Staff
  • Posts: 408
  • Joined: Apr 18, 2016
|
#30148
Conclusion: Lack of coverage for laboratory tests :arrow: quality of medical care decreased

If you'd like to diagram this question, you want to start with the sufficient from the conclusion and end with the necessary, and see where you think there is a gap.

Lack of coverage for laboratory tests :arrow: laboratory tests not done ...
accuracy of diagnosis decreased :arrow: quality of medical care decreased
The missing link is between not doing the laboratory tests and the accurate diagnosis.

So, I look for an answer choice that includes "laboratory tests not done" and "accuracy of diagnosis."
(A) Physical examinations and the uncovered laboratory tests together provide a more accurate diagnosis of many diseases than do physical examinations alone.
 LustingFor!L
  • Posts: 80
  • Joined: Aug 27, 2016
|
#34612
In explanation above, why would we use an assumption negation technique on a justify question?
User avatar
 Dave Killoran
PowerScore Staff
  • PowerScore Staff
  • Posts: 5853
  • Joined: Mar 25, 2011
|
#35447
I deleted the section where she mentioned that because it was confusing. It's a tricky and complex point under the best of circumstances, and even when an explanatory comment was added it wasn't clear enough in my opinion, so I killed that too :-D

  • The basic rule is: don't use the Assumption Negation Technique on a Justify question. It won't work reliably, and it takes too much time to figure out when it might or might work.

So, what was happening then, and why did she mention it? In a Justify question, the answer is sometimes simply an assumption of the argument. In those cases, the Negation Technique would work. Problem is, in many other situations, the correct answer in a Justify question is an assumption plus a bit more (and that extra part is not an assumption). In those cases, the Negation technique is not as reliable. Since I don't like to recommend techniques that only work some of the time (and you can't tell when ), the best practice is to not use the Negation Technique in these questions.

Apologies for the confusion, but hopefully that fixes it!
 cindyhylee87
  • Posts: 29
  • Joined: May 21, 2017
|
#36929
Hi,

I wondered why (B) couldn't justify the conclusion. (B) actually mentioned that MANY physicians limit the use of lab tests, and this seems to me that the gap between premise and conclusion has been closed because if lab test + physical exam would lead to more accurate diagnosis, then limiting the use of lab test would actually decrease the quality of medical care provided to patients(the diagnosis might not be accurate without lab test).

Thanks,
Cindy
 mariahenain
  • Posts: 30
  • Joined: Jun 09, 2017
|
#36978
Hi,

I'm having some trouble with Justify questions as a whole, specifically because I'm finding it difficult to apply the Justify Formula to most of the questions. How can I apply it to this question? The diagram illustrated above is a little confusing to me.
 cindyhylee87
  • Posts: 29
  • Joined: May 21, 2017
|
#37365
Hi Powerscore,

I have posted my question for 10 days, but no reply yet.

Thank you,
Cindy
User avatar
 Dave Killoran
PowerScore Staff
  • PowerScore Staff
  • Posts: 5853
  • Joined: Mar 25, 2011
|
#37378
Hi Cindy,

Thanks for the note! I see what happened here—you posted and then another user followed on, which made it appear to us that the question wasn't open (it's rare but can happen when the posting volume is very high, as it has been recently). We'll post an answer for you today.

Thanks!
 Jon Denning
PowerScore Staff
  • PowerScore Staff
  • Posts: 904
  • Joined: Apr 11, 2011
|
#37383
Hi Cindy and Maria! Let me take a stab at explaining this one in hopes that it clears up any lingering confusion (I'll address your specific questions after I run through the stimulus and correct answer) :)

This is a Justify the Conclusion question, so let's start by identifying the conclusion that we need to prove. The author is arguing that insurance company regulations that don't cover certain lab tests decrease the quality of medical care provided to patients. In other words, without having certain tests covered by insurance patients aren't getting as high a quality of medical care as they could be getting. Why? Because, from the first sentence, lab tests are often employed (along with physical exams) in order to diagnose diseases accurately.

So what we need to show then in order to prove the author's conclusion is that the uncovered tests are indeed useful for more accurately diagnosing diseases. That would show that failing to receive those tests would decrease the quality of medical care patients are getting (it's safe to say that diagnosing diseases less accurately is going to have harmful medical effects compared to more accurate diagnoses).

And that's what I'd be looking for in an answer!

So which answer gives us that information? Answer choice (A): with those uncovered lab tests included in the treatment process the accuracy of disease diagnoses increases. Improved accuracy of diagnosis would clearly be a good thing for patients, so when those helpful tests aren't covered (and thus fewer people get them; that's what covered by insurance means, after all) then the conclusion here would be correct: medical care quality would decrease.

Cindy you've asked about answer choice (B) and why it doesn't Justify. The answer is that (B) tells us nothing about those uncovered tests being useful/helpful or not, so we can't know that including them or skipping them would have any effect on the quality of care provided. It doesn't matter if many physicians are opposed to the tests not being covered. All we care about is whether those uncovered tests would improve the quality of med care or not. (A) indicates that they would. (B) doesn't say.

Maria your question is a bit broader, where you're wondering how the Justify Formula applies to this question (and perhaps how to better employ it in general). First, this isn't a question I would attempt to diagram. There's really nothing in it that screams "conditional connectivity" to me, which is typically what triggers the need to start sketching out the pieces. Instead, I would simply think about how to get from the premise (some lab tests are used to help diagnose diseases accurately) to the conclusion (when some lab tests aren't covered by insurance the quality of med care is lowered). That's all the Justify Formula really is: what can I add as an additional piece of information that will take me from what I already have as evidence (premise) to this author's belief (conclusion). Here all we need to do is show that the uncovered tests would lead to more accurate diagnoses, and then we'd know that failing to receive them (which is what would happen in a lot of cases if they aren't covered by insurance) would potentially be harmful (which is true if the accuracy of disease diagnoses drops). So when I think of the Justify Formula I'm not necessarily thinking about it in terms of diagrams or conditional sketches, where we have precise gaps to fill, but more in terms of movement, and how I can get from premise to conclusion with confidence.

I hope that helps!

Get the most out of your LSAT Prep Plus subscription.

Analyze and track your performance with our Testing and Analytics Package.